9
$\begingroup$

I know that there are no solutions to $2^n\equiv 1\pmod{n}$ for $n>1$ and I can prove that there are infinitely many $n$ such that $2^{n+1}\equiv1\pmod{n}$.
My question is:

Do we know other fixed values $k\in \mathbb{N}$ such that $2^{n+k}\equiv 1\pmod{n}$ holds infinitely often?

$\endgroup$
2
  • $\begingroup$ I believe any odd $k$ works, but I don't have a reference nor a proof at hand. $\endgroup$ Dec 19, 2017 at 18:13
  • $\begingroup$ Any $k\geq 1$ works, see my proof below. $\endgroup$
    – GH from MO
    Dec 19, 2017 at 22:13

2 Answers 2

10
$\begingroup$

For any $k\geq 1$, there are infinitely many solutions of the congruence $2^{n+k}\equiv 1\pmod{n}$. To see this, observe first that there is always a solution $n\geq 1$ satisfying $n+k\geq 7$. Indeed, for $k\geq 6$ this is verified by the trivial solution $n=1$, while for $1\leq k\leq 5$ it is verified by the pairs $$ (k,n) \ = \ (1,15),\ \ (2,7),\ \ (3,5),\ \ (4,31),\ \ (5,3).$$ Now it suffices to show that, for any fixed $k$ and for any solution $n\geq 1$ satisfying $n+k\geq 7$, there is a bigger solution $n'>n$ for the same $k$. Indeed, let $p$ be a primitive prime divisor of $2^{n+k}-1$, which exists by Zsigmondy's theorem. Then, the order of $2$ modulo $p$ equals $n+k$, whence $n+k\mid p-1$. This implies that $pn+k=(p-1)n+(n+k)$ is divisible by $n+k$, hence $$p,n\mid 2^{n+k}-1\mid 2^{pn+k}-1.$$ However, $p>n+k>n$, so $p$ and $n$ are coprime, and the above implies that $pn\mid 2^{pn+k}-1$. That is, $n'=pn$ is a solution bigger than $n$.

$\endgroup$
1
  • 2
    $\begingroup$ Very nice proof. My suspicion that it was true only for odd k came from experimentation: for instance for $k=6$, the first is $n=127$, then $n=3263257$, but by pushing further I found a few more. $\endgroup$ Dec 19, 2017 at 22:35
8
$\begingroup$

Sorry, this is in fact trivial. For instance if $2^{n+1}\equiv1\pmod{n}$ then $n$ is of course odd, so $2^{3n+3}\equiv1\pmod{n}$ and mod $3$, so $2^{3n+3}\equiv1\pmod{3n}$. This probably works for all odd $k$ not only $3$.

$\endgroup$
6
  • 4
    $\begingroup$ In addition to being odd, $n$ must be co-prime with $3$, right? $\endgroup$
    – Seva
    Dec 19, 2017 at 19:03
  • $\begingroup$ no, wrong. n odd is sufficient $2^2\equiv1\pmod{3}$. $\endgroup$ Dec 19, 2017 at 19:26
  • 1
    $\begingroup$ I certainly agree that $2^2\equiv1\pmod 3$, but if $3\mid n$, then $K\equiv 1\pmod n$ and $K\equiv 1\pmod 3$ do not imply $K\equiv 1\pmod{3n}$ - at least in the absence of other information. $\endgroup$
    – Seva
    Dec 19, 2017 at 19:33
  • 2
    $\begingroup$ But it does imply $K^{3} \equiv 1 (\mod 3n)$. $\endgroup$ Dec 19, 2017 at 20:21
  • $\begingroup$ @RobertIsrael: I see, it does work this way. $\endgroup$
    – Seva
    Dec 19, 2017 at 20:50

Your Answer

By clicking “Post Your Answer”, you agree to our terms of service and acknowledge you have read our privacy policy.

Not the answer you're looking for? Browse other questions tagged or ask your own question.